Calculating the Swing Height of a Thin Rod After a Collision with Putty

In summary: L^2}{4}+\frac{ML^2}{3})(\frac{vm}{2L(\frac{m}{4}+\frac{M}{3})})^2 = (m+M)gh/2 Solving for h will give you the height of the bottom of the rod. :smile:In summary, to find the height of the bottom of a thin rod after a collision with a mass m of putty, one must use the equation (\frac{mL^2}{4}+\frac{ML^2}{3})(\frac{vm}{2L(\frac{m}{4}+\frac{M}{3})})^2 = (m
  • #1
toothpaste666
516
20

Homework Statement



A thin rod of mass M and length L is suspended vertically from a frictionless pivot at its upper end. A mass m of putty traveling horizontally with a speed v strikes the rod at its CM and sticks there. How high does the bottom of the rod swing?

Homework Equations


The Attempt at a Solution



First I must look at the collision:

[itex] v = ωR = ω\frac{L}{2} [/itex]

[itex] ω = \frac{2v}{L} [/itex]

[itex] I_m = mR^2 = m(\frac{L}{2})^2 = \frac{mL^2}{4} [/itex]

[itex] I_M = \frac{1}{12}ML^2 [/itex]

using angular momentum:

[itex] I_m (\frac{2v}{L}) = (I_m + I_M)ω [/itex]

[itex] \frac{mL^2}{4} (\frac{2v}{L}) = (\frac{mL^2}{4} + \frac{1}{12}ML^2)ω [/itex]

[itex] \frac{mLv}{2} = (\frac{mL^2}{4} + \frac{1}{12}ML^2)ω [/itex]Have I set this up right so far? I am trying to find the angular velocity at the end of the collision, convert it to linear velocity, and plugging that into an energy equation
 
Physics news on Phys.org
  • #2
toothpaste666 said:

Homework Statement



A thin rod of mass M and length L is suspended vertically from a frictionless pivot at its upper end. A mass m of putty traveling horizontally with a speed v strikes the rod at its CM and sticks there. How high does the bottom of the rod swing?

Homework Equations


The Attempt at a Solution



First I must look at the collision:

[itex] v = ωR = ω\frac{L}{2} [/itex]

[itex] ω = \frac{2v}{L} [/itex]

[itex] I_m = mR^2 = m(\frac{L}{2})^2 = \frac{mL^2}{4} [/itex]

[itex] I_M = \frac{1}{12}ML^2 [/itex]

using angular momentum:

[itex] I_m (\frac{2v}{L}) = (I_m + I_M)ω [/itex]

[itex] \frac{mL^2}{4} (\frac{2v}{L}) = (\frac{mL^2}{4} + \frac{1}{12}ML^2)ω [/itex]

[itex] \frac{mLv}{2} = (\frac{mL^2}{4} + \frac{1}{12}ML^2)ω [/itex]Have I set this up right so far? I am trying to find the angular velocity at the end of the collision, convert it to linear velocity, and plugging that into an energy equation
You need to use a different formula for the moment of inertia of the rod. Here, the rod is pivoting at its end, not at its center.

Other than that, it looks good so far. :smile:
 
  • #3
So it would be this instead?

[itex] \frac{mLv}{2} = (\frac{mL^2}{4} + \frac{1}{3}ML^2)ω [/itex]
 
  • #4
toothpaste666 said:
So it would be this instead?

[itex] \frac{mLv}{2} = (\frac{mL^2}{4} + \frac{1}{3}ML^2)ω [/itex]
Yeah, that looks right. :approve:
 
  • #5
ok so:


[itex] \frac{v}{2} = (\frac{L}{4} + \frac{L}{3})ω [/itex]


[itex] \frac{v}{2} = \frac{7L}{12} ω [/itex]

[itex] v = \frac{14L}{12} ω [/itex]

[itex] v = \frac{7L}{6} ω [/itex]

[itex] ω = \frac{6v}{7L} [/itex]

to get linear velocity use v1=Rω

[itex] v_1 = (\frac{L}{2}) \frac{6v}{7L} [/itex]

[itex] v_1 = \frac{3v}{7} [/itex]


plugging this into an energy equation:


[itex].5(m+M)(\frac{3v}{7})^2 = (m+M)gh[/itex]

is this the correct set up for the energy equation to find h?
 
  • #6
wait i canceled out masses where I couldn't have. one sec
 
  • #7
[itex] \frac{vm}{2} = (\frac{mL}{4} + \frac{ML}{3})ω [/itex]

[itex] \frac{vm}{2L} = (\frac{m}{4} + \frac{M}{3})ω [/itex]

[itex] \frac{vm}{2L(\frac{m}{4} + \frac{M}{3})} = ω [/itex]



to get linear velocity use v1=Rω

[itex] v_1 = (\frac{L}{2}) \frac{vm}{2L(\frac{m}{4} + \frac{M}{3})} [/itex]

[itex] v_1 = \frac{vm}{4(\frac{m}{4} + \frac{M}{3})} [/itex]


plugging this into an energy equation:


[itex].5(m+M)(\frac{vm}{4(\frac{m}{4} + \frac{M}{3})})^2 = (m+M)gh[/itex]

is this the correct set up for the energy equation to find h?
 
  • #8
toothpaste666 said:
[itex] \frac{vm}{2} = (\frac{mL}{4} + \frac{ML}{3})ω [/itex]

[itex] \frac{vm}{2L} = (\frac{m}{4} + \frac{M}{3})ω [/itex]

[itex] \frac{vm}{2L(\frac{m}{4} + \frac{M}{3})} = ω [/itex]
I'm good with everything up to this point. :approve:

to get linear velocity use v1=Rω

[itex] v_1 = (\frac{L}{2}) \frac{vm}{2L(\frac{m}{4} + \frac{M}{3})} [/itex]

[itex] v_1 = \frac{vm}{4(\frac{m}{4} + \frac{M}{3})} [/itex]


plugging this into an energy equation:


[itex].5(m+M)(\frac{vm}{4(\frac{m}{4} + \frac{M}{3})})^2 = (m+M)gh[/itex]

is this the correct set up for the energy equation to find h?

But I don't think that's quite right. I wouldn't convert to linear velocity just yet. It's not really valid since different parts of the rod are moving at different speeds, and it's not valid to model the kinetic energy of the rod simply by the motion of its center of mass; it has rotational energy too.

Instead, determine the initial kinetic energy in the form of [itex] \frac{1}{2}I \omega^2 [/itex] There's no reason to move away from angular coordinates just yet.

You then have the choice of how you want to represent the final height and the final potential energy. Above, you used [itex] (m + M)gh [/itex], which is fine. But realize here that h is the change in height of the system's center of mass. The problem asks you to find the final height of the bottom of the rod, not the center of mass. So you'll have to double h before submitting your final answer.
 
  • #9
ok so the energy equation would be :

[itex] (\frac{mL^2}{4}+\frac{ML^2}{3})(\frac{vm}{2L(\frac{m}{4}+\frac{M}{3})})^2 = (m+M)2gh [/itex]


??
 
  • #10
toothpaste666 said:
ok so the energy equation would be :

[itex] (\frac{mL^2}{4}+\frac{ML^2}{3})(\frac{vm}{2L(\frac{m}{4}+\frac{M}{3})})^2 = (m+M)2gh [/itex]


??

Yes. At least that's the same equation that I got. :approve:

Of course, some of the terms cancel, which makes it a little easier.

So solve for h, double it, and I think you're done. :smile:
 
  • #11
ahh so making it 2h wouldn't make it right? i should have left it as h, solved for h and THEN doubled it?
 
  • #12
toothpaste666 said:
ahh so making it 2h wouldn't make it right? i should have left it as h, solved for h and THEN doubled it?

Well, I thought your 2h on the right side came from the 1/2 in the [itex] \frac{1}{2}I \omega^2 [/itex] on the left side (it seemed to me that you multiplied both sides of the equation by 2).

But yes, solving for h and then doubling it at the end should work. If instead you want to define h as the final height of the end of the rod (not the center of mass), from the get-go, the potential energy would be (m + M)g(h/2), would it not?
 
  • #13
ah right i forgot about that 1/2 by accident. so the 2 would cancel out with the h/2 leaving

[itex] (\frac{mL^2}{4}+\frac{ML^2}{3})(\frac{vm}{2L(\frac{m}{4}+\frac{M}{3})})^2 = (m+M)gh [/itex]

and solving for h here would give me the height?
 
  • #14
toothpaste666 said:
ah right i forgot about that 1/2 by accident. so the 2 would cancel out with the h/2 leaving

[itex] (\frac{mL^2}{4}+\frac{ML^2}{3})(\frac{vm}{2L(\frac{m}{4}+\frac{M}{3})})^2 = (m+M)gh [/itex]

and solving for h here would give me the height?

Yes, I believe that is correct. No need to double h if you use the above equation.
 
  • Like
Likes 1 person
  • #15
thank you!
 

Related to Calculating the Swing Height of a Thin Rod After a Collision with Putty

1. What is Angular Momentum?

Angular momentum is a measure of an object's rotational motion. It is defined as the product of an object's moment of inertia and its angular velocity. In simpler terms, it is the amount of rotational motion an object possesses.

2. How is Angular Momentum calculated?

The formula for calculating angular momentum is L = Iω, where L is angular momentum, I is moment of inertia, and ω is angular velocity. Moment of inertia is a measure of an object's resistance to rotational motion and is dependent on the mass and distribution of the object's mass.

3. What is Conservation of Angular Momentum?

Conservation of angular momentum states that the total angular momentum of a system remains constant unless acted upon by an external torque. This means that in a closed system, the total amount of angular momentum will remain the same, even if individual objects within the system may experience changes in their own angular momentum.

4. How does Angular Momentum relate to Energy?

Angular momentum and energy are closely related, as they are both conserved quantities in a closed system. In rotational motion, kinetic energy is directly proportional to angular velocity, and angular momentum is directly proportional to both moment of inertia and angular velocity. This means that changes in one quantity will affect the other.

5. What are some real-world examples of Angular Momentum?

Some examples of angular momentum in everyday life include spinning tops, figure skaters performing spins, and a planet orbiting around the sun. In each of these examples, the objects possess angular momentum due to their rotational motion. Another example is a rolling ball, where both linear and angular momentum are present due to the translational and rotational motion of the ball.

Similar threads

  • Introductory Physics Homework Help
Replies
12
Views
1K
  • Introductory Physics Homework Help
Replies
10
Views
967
  • Introductory Physics Homework Help
Replies
11
Views
1K
  • Introductory Physics Homework Help
10
Replies
335
Views
8K
  • Introductory Physics Homework Help
2
Replies
55
Views
2K
  • Introductory Physics Homework Help
Replies
10
Views
1K
  • Introductory Physics Homework Help
Replies
10
Views
3K
  • Introductory Physics Homework Help
Replies
21
Views
1K
  • Introductory Physics Homework Help
Replies
16
Views
2K
  • Introductory Physics Homework Help
Replies
12
Views
2K
Back
Top